LSAT and Law School Admissions Forum

Get expert LSAT preparation and law school admissions advice from PowerScore Test Preparation.

 jcough346
  • Posts: 35
  • Joined: Aug 05, 2016
|
#31388
Im having difficulty understanding why A is correct. If a "large number" cant get there then that seems like a pretty good reason to shut it down thereby weakening the argument to keep the rec center open...
 Adam Tyson
PowerScore Staff
  • PowerScore Staff
  • Posts: 5153
  • Joined: Apr 14, 2011
|
#31405
I think you might be misreading that one, jcough. We are talking about closing the facility that is in their neighborhood. A says that a large number of residents cannot get to recreation facilities outside their locality. That means that closing the one that is inside their locality would essentially deny them access to any recreational facility. That's a big strengthen answer! The other answers all weaken the argument, showing one way or another that local residents aren't using or don't need their neighborhood facility.
 tetsuya0129
  • Posts: 73
  • Joined: Jun 20, 2018
|
#83656
Hi Powerscore staff,

I picked and agree with the credited answer; I am only writing to discuss what's the weakening rationale behind (E), which gave me a "WTF" during my practice.

(E) tells us recreation centers are becoming less and less important; yet, the centers can still be a necessity for people in a neighborhood even though less important.

How come the concept of necessity (a necessary thing) entails importance?

Much appreciated for your time and help,
Leon
 Adam Tyson
PowerScore Staff
  • PowerScore Staff
  • Posts: 5153
  • Joined: Apr 14, 2011
|
#83978
Answer E weakens the argument by contradicting one of the premises, Leon. One resident complained that the recreation center is necessary, and that closing it is therefore unacceptable, but answer E is saying that resident may be wrong, as the necessity for a recreation center is declining. So perhaps closing the rec center could be acceptable!

In a Weaken-EXCEPT question it is common for some of the answers to attack the premises, because without the premises to hold up the conclusion, the conclusion isn't very strong.
 tetsuya0129
  • Posts: 73
  • Joined: Jun 20, 2018
|
#86576
Thank you, Adam!

Get the most out of your LSAT Prep Plus subscription.

Analyze and track your performance with our Testing and Analytics Package.